Themenbereiche Themenbereiche Profile Hilfe/Anleitungen Help    
Recent Posts Last 1|3|7 Days Suche Suche Tree Tree View  

X hoch x hoch x hoch...

ZahlReich - Mathematik Hausaufgabenhilfe » Universitäts-Niveau » Analysis » Konvergenz » X hoch x hoch x hoch... « Zurück Vor »

Das Archiv für dieses Kapitel findest Du hier.

Autor Beitrag
Seitenanfangvoriger Beitragnächster BeitragSeitenende Link zu diesem Beitrag

Martin
Suche alle Beiträge dieser Person in dieser Hauptrubrik
Veröffentlicht am Sonntag, den 22. Juli, 2001 - 09:19:   Beitrag drucken

Es sei x > 0 und die Folge xn festgesetzt durch x1 = x, x2 = xx1, x3 = xx2, ..., xn = xxn-1. (Das xn im Exponenten steht für xn, die Formatierung kann es leider nicht korrekt darstellen.)

Für welche x fällt diese Folge konvergent aus?

Warum lautet die Antwort:
(1/e)e £ x £ e1/e ?

Wie beweißt man das?

Bitte helft mir!

Martin.
Seitenanfangvoriger Beitragnächster BeitragSeitenende Link zu diesem Beitrag

H.R.Moser,megamath.
Suche alle Beiträge dieser Person in dieser Hauptrubrik
Veröffentlicht am Sonntag, den 22. Juli, 2001 - 19:52:   Beitrag drucken

Hi Martin,

Da ich in Kürze in die Ferien II nach Griechenland
abreisen werde, kann ich mich Deiner neuesten Aufgabe
nur ganz summarisch und in einem abgekürzten Verfahren
widmen.
Vielleicht springt sonstwer in die Lücke und sorgt
für die nötigen Ergänzungen.

Zwischen einem allenfalls vorhandenen Grenzwert g
und der Ausgangszahl x besteht die Relation
g = x ^ g., also x = g ^ (1 / g)
Wir betrachten die Funktion f in der unabhängigen Variablen g
x = f(g) = g ^ (1/g) für g > 0 und ermitteln ihr Maximum M.
Wir berechnen die Ableitung f '(g) durch logarithmisches
Differenzieren nach g.
Aus ln ( f(g)) = 1 / g * l n g folgt
f ' / f = - 1 / g^2 * ln g + 1 / g^2 oder
f ' (g) = f / g^2 * [ 1 - ln g ]
Nullstelle der Ableitung: g = e; hier liegt ein relatives Maximum vor,
welches zugleich ein absolutes Maximum ist.
Für M erhalten wir den Wert:
M = e ^(1/e) ;
°°°°°°°°°°°°
dieser Wert erscheint auch in der Aufgabenstellung
als obere Grenze der x-Werte
etc.

Mit freundlichen Grüßen
H.R.Moser,megamath.
Seitenanfangvoriger Beitragnächster BeitragSeitenende Link zu diesem Beitrag

Zaph (Zaph)
Suche alle Beiträge dieser Person in dieser Hauptrubrik
Veröffentlicht am Montag, den 23. Juli, 2001 - 19:59:   Beitrag drucken

Hallo Martin,

Megamath hat dir gezeigt, dass x <= e^(1/e) eine notwendige Bedingung für die Konvergenz der Folge ist.

Dass (1/e)^e <= x <= e^(1/e) hinreichend ist, könnte mit dem Banachschen Fixpunktsatz funktionieren. Kennst du den? Habe ich aber noch nicht vollständig durchgedacht.

Bliebe dann noch die Notwendigkeit von x >= (1/e)^e zu zeigen.
Seitenanfangvoriger Beitragnächster BeitragSeitenende Link zu diesem Beitrag

Martin
Suche alle Beiträge dieser Person in dieser Hauptrubrik
Veröffentlicht am Dienstag, den 24. Juli, 2001 - 10:44:   Beitrag drucken

Kann man nicht, wie Megamath es schon mit e1/e als Maximum tat, (1/e)e als absolutes Minimum bestimmen?

Muß man überhaupt zeigen, daß die Bedingung hinreichend ist?

Martin.

P.S: Am wichtigsten ist eigentlich die Beantwortung der ersten Frage.
Seitenanfangvoriger Beitragnächster BeitragSeitenende Link zu diesem Beitrag

Zaph (Zaph)
Suche alle Beiträge dieser Person in dieser Hauptrubrik
Veröffentlicht am Dienstag, den 24. Juli, 2001 - 17:34:   Beitrag drucken

Megamath hat die Funktion f(g) = g^(1/g) betrachtet, weil die Folge nur dann konvergieren kann, wenn es ein g gibt mit f(g) = x.

Aber wenn es solch ein g gibt, ist es noch lange nicht klar, dass die Folge konvergiert. Es ist lediglich NOTWENDIG.

Megamath hat dann gezeigt, dass f(g) immer kleiner oder gleich e^(1/e) ist, also für die Konvergenz NOTWENDIG ist, dass x <= e^(1/e).

Tatsächlich gibt es zu JEDEM x <= e^(1/e) ein g mit f(g) = x. Auch dann, wenn x < (1/e)^e. Es gilt nämlich f(g) -> 0 für g -> 0.

Um die x-Werte kleiner als (1/e)^e auszuschließen, musst du dir also etwas anderes einfallen lassen.

Jetzt zur wichtigsten Frage.

Wieso ist die Folge konvergent, wenn (1/e)^e <= x <= e^(1/e) ?

Wieso ist also (1/e)^e <= x <= e^(1/e) für die Konvergenz der Folge HINREICHEND?

Dazu noch einmal meine Frage an dich: Kennst du den Banachschen Fixpunktsatz? Was anderes fällt mir nämlich im Moment nicht ein.

Ich finde die Aufgabe sehr interessant. Ist dies eine ganz normale Aufgabe vom Analysis-Übungszettel?
Seitenanfangvoriger Beitragnächster BeitragSeitenende Link zu diesem Beitrag

Martin
Suche alle Beiträge dieser Person in dieser Hauptrubrik
Veröffentlicht am Dienstag, den 24. Juli, 2001 - 20:39:   Beitrag drucken

So viel ich weiß ist der Banachsche Fixpunktsatz nur ein rein theoretisches Hilfsmittel, mit dem man in konkreten Fällen nicht viel anfangen kann.

Mir fällt nur noch ein daß, wenn man (1/e)e und e1/e vergleicht, jeweils die Basis der einen Zahl der Kehrwert der Basis der anderen Zahl ist. Genauso verhält es sich beim Exponenten.

Das große Plus von Megamaths Beweis ist, daß er absolut konstruktiv vorgeht, daß er nicht einfach zeigt, daß die obere Schranke halt einfach passt, sondern daß er wirklich herleitet, warum die obere Schranke diese sein muß und keine andere.
Ich bezweifle, daß man mit dem Banachschen Fixpunktsatz konstruktiv zeigen kann, daß (1/e)e die kleinste untere Schranke ist, falls man es mit diesem Mittel überhaupt zeigen kann.
Außerdem stammt diese Aufgabe so viel ich weiß aus einer Zeit, in der es noch gar keinen Banachschen Fixpunktsatz gab.

Martin.
Seitenanfangvoriger Beitragnächster BeitragSeitenende Link zu diesem Beitrag

Zaph (Zaph)
Suche alle Beiträge dieser Person in dieser Hauptrubrik
Veröffentlicht am Dienstag, den 24. Juli, 2001 - 22:58:   Beitrag drucken

Ja, irgendwie sieht das wunderschön symmetrisch aus: Basis und Exponent jeweils der Kehrwert. Aber ich gleube, der Schein trügt. Hast du mal mit einem programmierbaren Rechner ausprobiert, was passiert, wenn x kleinere Werte als (1/e)^e bzw. größere Werte als e^(1/e) annimmt? Schon witzig ...

Nein, dass (1/e)^e die größte untere Schranke ist, kann man mit dem BFS bestimmt nicht zeigen.

Habe ich auch nicht behauptet. Im Gegenteil: Ich habe gesagt, dass man ZUSÄTZLICH die Notwendigkeit von x >= (1/e)^e beweisen muss.

Aber dass der BFS nicht konstruktiv ist, stimmt nicht. Sagt er doch im speziellen folgendes:

Es sei h: I ---> I eine Abbildung (I ein abgeschlossenes Ihtervall) und a eine reelle Zahl mit 0 < a < 1, sodass

|h(s) - h(t)| <= a*|s - t|

für alle s,t aus I erfüllt ist.

Dann besitzt h einen Fixpunkt f aus I. Wenn x0 aus I beliebig gewählt ist und xn+1 = h(xn), dann konvergiert die Folge (xn) gegen f.


Betrachte hier die Abbildung h(t) = x^t auf dem Intervall I = [(1/e)^e,e^(1/e)] und x0 = x.

Dann gilt schonmal, dass h eine Abbildung von I nach I ist. (Beweis!)

Leider ist der Rest der Voraussetzungen des BFS nicht vollständig erfüllt. Zumindest gilt aber

|h(s) - h(t)| < |s - t|

für alle s,t aus I mit s != t. (Beweis!)

P.S.: Dass die Aufgabe älter als der BFS ist, sagt doch nicht, dass man die Aufgabe nicht mit dem BFS lösen kann.
Seitenanfangvoriger Beitragnächster BeitragSeitenende Link zu diesem Beitrag

Zaph (Zaph)
Suche alle Beiträge dieser Person in dieser Hauptrubrik
Veröffentlicht am Dienstag, den 24. Juli, 2001 - 23:10:   Beitrag drucken

Quatsch, das Intervall soll lauten

I = [1/e , e]
Seitenanfangvoriger Beitragnächster BeitragSeitenende Link zu diesem Beitrag

Zaph (Zaph)
Suche alle Beiträge dieser Person in dieser Hauptrubrik
Veröffentlicht am Mittwoch, den 25. Juli, 2001 - 21:59:   Beitrag drucken

Hallo Martin,

noch interessiert?

Vergiss das mal wieder, was ich gerade gesagt habe. Zumindest alles das, was nach dem grünen kommt.

Wenn x = 1, dann ist klar, dass die Folge konvergiert.

Sei nun 1 < x <= e^(1/e).

Mit vollständiger Induktion ist nun leicht zu zeigen, dass

1 < xn < e

für alle n erfüllt ist. Folglich ist die Folge monoton wachsend und nach oben beschränkt, also konvergent.

Bleibt der Fall x < 1 ...

Die Folge ist Experimenten zufolge nicht mehr monoton. Wenn x >= (1/e)^e, scheinen die Folgeglieder abwechselnd größer und kleiner als der Grenzwert zu sein. Wenn 0 < x < (1/e)^e, dann hat die Folge wohl zwei Häufungswerte. Einer ist größer und der andere ist kleiner als das g mit g^(1/g) = x.
Seitenanfangvoriger Beitragnächster BeitragSeitenende Link zu diesem Beitrag

Martin
Suche alle Beiträge dieser Person in dieser Hauptrubrik
Veröffentlicht am Donnerstag, den 26. Juli, 2001 - 13:03:   Beitrag drucken

Meinst Du, die Aufgabe könnte insgesamt mithilfe von Häufungswertbetrachtungen gelöst werden? Wäre diese Möglichkeit nicht eine Überlegung deinerseits wert?
Tut mir leid, ich kenne mich halt auf dem Gebiet kaum aus; die Aufgabe interessiert mich aber trotzdem.

Martin.

P.S: Gestern kam kein Posting, weil ich Probleme mit meinem Internetanschluß hatte.
Seitenanfangvoriger Beitragnächster BeitragSeitenende Link zu diesem Beitrag

Hans (Birdsong)
Suche alle Beiträge dieser Person in dieser Hauptrubrik
Veröffentlicht am Donnerstag, den 26. Juli, 2001 - 17:43:   Beitrag drucken

Hallo:

Hier ein paar Ideen. Ich benutze aber andere Bezeichnungen: Sei a>0, a<>1,

f(x) := a^x (f haengt also von a ab).

Die Iterationsvorschrift lautet also x_0 = 1,
x_(n+1)= f(x_n) , speziell x_1 = a. Betrachte
den Graphen C : y = f(x) und die Gerade g : y = x.
Wenn lim x_n = c existiert, so ist (c,c) ein
Schnittpunkt von C und g. Bestimmt man dasjenige
C, welches g tangiert (offenbar Grenzfall fŸr die Konvergenz), so findet man leicht
a = exp(1/e). FŸr a > exp(1/e) haben C und g
keinen Schnittpunkt, fŸr

(1) 1 < a =< exp(1/e)

haben wir 2 Schnittpunkte, d.h. 2 Loesungen
c_1,c_2 der Gl. c = f(c), mit c_1 = c_2 = e g.d.w.
a = exp(1/e).
Wir zeigen, dass (x_n) streng wachsend gegen c_1
konvergiert.

1.f ist wachsend, also 1 = x_0 < c_1. Sei
x_n < c_1 schon gesichert, dann ist x_(n+1) =
f(x_n) < f(c_1) = c_1. Also x_n < c_1 fŸr alle n.

2. FŸr 0 < x < c_1 gilt f(x) > x ==> x_(n+1) =
f(x_n) > x_n.

Nun sei 0 < a < 1. Dann ist f fallend, es gibt
genau 1 Fixpunkt c.Der Grenzfall fŸr die Konvergenz tritt analog zum obigen offenbar dann ein, wenn C in c die Steigung -1 hat, g also Normale ist. HierfŸr findet man leicht die Bedingung a = (1/e)^e, c = 1/e. Jetzt sollte man noch zeigen, dass fŸr a > (1/e)^e wirklich
lim x_n = c ist.

mfg

Hans
Seitenanfangvoriger Beitragnächster BeitragSeitenende Link zu diesem Beitrag

Martin
Suche alle Beiträge dieser Person in dieser Hauptrubrik
Veröffentlicht am Donnerstag, den 26. Juli, 2001 - 19:27:   Beitrag drucken

Allerherzlichsten Dank für deine Ideen Hans!
Vielleicht bekommst Du den letzten Teil des Beweises ja auch noch hin. :-)

Martin.
Seitenanfangvoriger Beitragnächster BeitragSeitenende Link zu diesem Beitrag

Hans (Birdsong)
Suche alle Beiträge dieser Person in dieser Hauptrubrik
Veröffentlicht am Freitag, den 27. Juli, 2001 - 21:48:   Beitrag drucken

Hallo Martin:

Noch ein Gedankensplitter. Wir betrachten den Fall

(1/e)^e < a < 1.

Es sei m:= f'(c). Eliminiert man c aus den beiden
Gleichungen

a^c = c und a^c*ln(a) = m,

so findet man

a = exp(m*e^(-m)) =: h(m).

Es gilt

h'(m) = (1+m)*e^(-m)*h(m) > 0,

also h streng wachsend, es existiert die Inverse

h^(-1), sie ist ebenfalls wachsend und bildet

](1/e)^e , 1[ auf ]-1,0[ ab. FŸr diese a ist

also |f'(c)| < 1 , wegen der Stetigkeit von f'

gilt also |f'(x)|<1 in einer Umgebung von c.

Nach einem einschlaegigen Satz konvergiert also

die Iterationsfolge (x_n) gegen c. Leider weiss

man damit noch nichts im Grenzfall a = (1/e)^e.

mfg

Hans
Seitenanfangvoriger Beitragnächster BeitragSeitenende Link zu diesem Beitrag

Martin
Suche alle Beiträge dieser Person in dieser Hauptrubrik
Veröffentlicht am Samstag, den 28. Juli, 2001 - 12:23:   Beitrag drucken

Hallo Hans,

ich habe gerade bemerkt, daß die Aufgabe schon einmal in aller Ausführlichkeit und Konsequenz im "Mathe macht Spaß"-Forum behandelt wurde.

Am Ende der Nachricht habe ich einen Link zum Beweis im oben genannten Forum angefügt.

Martin.

Beweis aus dem "Mathe macht Spaß"-Forum

Beitrag verfassen
Das Senden ist in diesem Themengebiet nicht unterstützt. Kontaktieren Sie den Diskussions-Moderator für weitere Informationen.

ad

Administration Administration Abmelden Abmelden   Previous Page Previous Page Next Page Next Page